GERO FINAL

Ace your homework & exams now with Quizwiz!

The nurse suspects that an older patient is experiencing Parkinsons disease. What did the nurse observe in this patient? Standard Text: Select all that apply. 1. Chorea 2. Tremor 3. Apraxia 4. Agnosia 5. Dystonia

1 2 5

A patient prescribed medications for gastroesophageal reflux disease (GERD) is at risk for altered absorption of which nutrients? Standard Text: Select all that apply. 1. Iron 2. Calcium 3. Folic acid 4. Vitamin D 5. Vitamin B12

1. Iron 2. Calcium 5. Vitamin B12

The nurse is instructing an older patient on the use of My Plate to ensure an adequate nutritional intake. Which food items would the nurse teach the patient to fill on one-half of the plate? Standard Text: Select all that apply. 1. Oils 2. Fruits 3. Grains 4. Proteins 5. Vegetables

2. Fruits 5. Vegetables

Which assessment data indicates to the nurse that an older patient is experiencing undernutrition? 1. Body mass index (BMI) of 20 2. Unintentional 3% weight loss over a month 3. Denial of taking a multiple vitamin supplement 4. Serum albumin slightly below normal, prealbumin and transferrin within normal limits

1. Body mass index (BMI) of 20

An older patient with a peptic ulcer asks why lifestyle alterations are needed. What should the nurse explain to the patient? 1. Alcohol stimulates gastric acid secretion. 2. Alcohol acts to suppress gastric immunity. 3. Caffeine is associated with abdominal pain. 4. Tobacco reduces the effectiveness of gastric ulcer medications.

1. Alcohol stimulates gastric acid secretion.

The nurse is teaching an older patient with diverticulitis on foods that could precipitate a painful attack. How should the nurse instruct this patient? 1. Avoid foods with seeds. 2. Restrict the intake of high fiber foods. 3. Limit the intake of eggs and dairy products. 4. Eat whole grains with sesame seeds for added protein.

1. Avoid foods with seeds.

The nurse is concerned that an older patient is experiencing dehydration. What did the nurse assess in this patient? Standard Text: Select all that apply. 1. Confusion 2. Headache 3. Weight gain 4. Long tongue furrows 5. Forearm tenting of the skin

1. Confusion 2. Headache 4. Long tongue furrows

Which gastrointestinal change in an older patient does the nurse recognize as being associated with aging? 1. Decreased esophageal motility 2. Decreased incidence of cholelithiasis 3. Increase in hydrochloric acid in the stomach 4. Increased absorption of nutrients in the intestines

1. Decreased esophageal motility

The nurse is using the DETERMINE nutrition screening tool when assessing the nutritional status of an older patient. Which criteria are included in this screening tool? Standard Text: Select all that apply. 1. Disease 2. Above the age of 80 3. Reduced social contact 4. Eats fruits and vegetables 5. Needs assistance in self-care

1. Disease 2. Above the age of 80 3. Reduced social contact 5. Needs assistance in self-care

The nurse is concerned that an older patient with dementia receiving psychotropic medications for sleep is experiencing side effects. What did the nurse assess in this patient? Standard Text: Select all that apply. 1. Dizziness 2. Constipation 3. Hallucinations 4. Daytime lethargy 5. Problems swallowing

1. Dizziness 2. Constipation 4. Daytime lethargy 5. Problems swallowing

An older patient is having difficulty sleeping. What can the nurse instruct the patient to help improve the patients sleep? Standard Text: Select all that apply. 1. Do not nap during the day. 2. Take a walk an hour before going to sleep. 3. Have a glass of wine before going to sleep. 4. Avoid reading or watching television in bed. 5. If unable to sleep, get up and go to another room.

1. Do not nap during the day. 4. Avoid reading or watching television in bed. 5. If unable to sleep, get up and go to another room.

Which observation made by the nurse suggests that a patient is having difficulty swallowing? 1. Drooling 2. Cheilosis 3. Long furrowed tongue 4. Unintentional weight loss

1. Drooling

What food items should the nurse teach an older patient to ingest to increase the dietary intake of vitamin D? 1. Eat liver at least once a week. 2. Plan to eat salmon at least twice a week. 3. Eat three servings of yogurt or cheese each day. 4. Red meat should be consumed every other day.

1. Eat liver at least once a week.

The nurse is concerned that an older patient is at risk for developing diverticulosis. What did the nurse assess in the patient? Standard Text: Select all that apply. 1. History of constipation 2. Low intake of dietary fiber 3. Intake high in protein and calcium 4. Diet high in refined carbohydrates 5. Physically inactive for many years

1. History of constipation 2. Low intake of dietary fiber 4. Diet high in refined carbohydrates 5. Physically inactive for many years

The nurse is assisting an older patient with dysphagia to eat an evening meal. Which foods on the patients tray should be avoided? Standard Text: Select all that apply. 1. Hot tea 2. Custard 3. Pudding 4. Milkshake 5. Clear broth

1. Hot tea 5. Clear broth

The nurse instructed an older patient on the importance of maintaining adequate hydration. Which statement by the patient indicates that additional teaching is needed? 1. Ill drink water and unsweetened beverages whenever I feel thirsty. 2. I can add an extra cup of decaffeinated coffee with breakfast and dinner. 3. I will set up a schedule to drink a glass of water every 2 hours throughout the day. 4. If I drink a lot of fluids, Ill have to go to the bathroom more often, but Ill get more exercise.

1. Ill drink water and unsweetened beverages whenever I feel thirsty.

What should the nurse instruct an older patient to do to support healthy eating habits? Standard Text: Select all that apply. 1. Increase fiber intake. 2. Reduce sodium intake. 3. Look for hidden sugar. 4. Enjoy olive oil and walnuts. 5. Complete a meal in 10 minutes.

1. Increase fiber intake. 2. Reduce sodium intake. 3. Look for hidden sugar. 4. Enjoy olive oil and walnuts.

Which interventions should the nurse use to reduce the risk of aspiration for an older patient with dysphagia? Standard Text: Select all that apply. 1. Monitor during meals for a change in respirations. 2. Maintain an upright position for 1 hour after eating. 3. Raise the head of the bed to a 90 degree angle during meals. 4. Provide pureed solid foods and thin clear liquids during meals. 5. Ensure that one bite has been swallowed before providing another.

1. Monitor during meals for a change in respirations. 2. Maintain an upright position for 1 hour after eating. 3. Raise the head of the bed to a 90 degree angle during meals. 5. Ensure that one bite has been swallowed before providing another.

The nurse is assessing an older patient who wakes up during the night. Which finding does the nurse identify as a risk factor for disturbed sleep? 1. Patient has osteoarthritis of both hips 2. Patient ingests one cup of coffee every morning 3. Patient takes antidepressant medication in the morning 4. Patient walks for half an hour before lunch each day

1. Patient has osteoarthritis of both hips

When planning care, for which older patients should the nurse identify as being at risk for malnutrition as a result of hypermetabolism? Standard Text: Select all that apply. 1. Patient with a fever 2. Patient with dysphagia 3. Patient with osteoporosis 4. Patient who is a vegetarian 5. Patient with chronic lung disease

1. Patient with a fever 5. Patient with chronic lung disease

The nurse is concerned that an older patient is experiencing dysphagia. What did the nurse assess in this patient? Standard Text: Select all that apply. 1. Slurred speech 2. Extreme lethargy 3. Talking while eating 4. Weak voice and cough 5. Drooling saliva from the mouth

1. Slurred speech 2. Extreme lethargy 4. Weak voice and cough 5. Drooling saliva from the mouth

The nurse is preparing an educational session on stroke prevention for a group of senior citizens. What information would be important for the nurse to provide to these participants? Standard Text: Select all that apply. 1. Stop smoking. 2. Limit exercise to once a week. 3. Maintain a healthy body weight. 4. Follow a low-sodium diet as prescribed. 5. Take blood pressure medication as prescribed

1. Stop smoking. 3. Maintain a healthy body weight. 4. Follow a low-sodium diet as prescribed. 5. Take blood pressure medication as prescribed.

An older patient with intermittent diarrhea has been advised to increase the amount of soluble fiber in the diet. What should the nurse suggest to this patient? 1. Use Metamucil as prescribed. 2. Restrict the intake of oranges. 3. Limit eating frozen vegetables. 4. Do not eat the peels of apples or pears.

1. Use Metamucil as prescribed.

An older patient is experiencing abdominal discomfort. What should the nurse do when examining this patients abdominal area? Standard Text: Select all that apply. 1. Warm the hands. 2. Begin with very light palpation. 3. Use moderate pressure on the painful area. 4. Palpate in areas farther away from the pain. 5. Begin the assessment with the area of most pain.

1. Warm the hands. 2. Begin with very light palpation. 4. Palpate in areas farther away from the pain.

During an assessment, the nurse determines that an older patient is taking several supplements that affect blood clotting. Which supplements did the nurse assess that this patient is taking? Standard Text: Select all that apply. 1. Zinc 2. Garlic 3. Fish oil 4. Ginseng 5. Vitamin E

2. Garlic 3. Fish oil 4. Ginseng 5. Vitamin E

The nurse instructs an older patient with Parkinsons disease about carbidopa-levodopa (Sinemet). Which patient statement indicates that teaching has been effective? 1. I will take the medication with my meals. 2. I will sit up on the side of the bed before standing. 3. This medication will cure my Parkinsons disease in time. 4. This medication will not affect my blood pressure medications.

2

An older patient is demonstrating signs of a brain attack. What will the nurse do to assess the degree of cerebral infarct? 1. Obtain a current cardiac rhythm strip. 2. Assess the patient using a stroke scale. 3. Prepare for an emergency electroencephalogram. 4. Obtain orders for immediate blood electrolyte analysis.

2.

An older patient with Alzheimers disease has a feeding tube. The family wants to know if the patient will ever be able to eat solid food again. What information should the nurse include when responding to this familys question? 1. The dietitian will decide if this can be done. 2. It depends upon the patients functional eating abilities. 3. This can be done but the feeding tube has to be removed first. 4. In the patient with dementia, the restoration of natural feeding is highly unlikely.

2.

An older patient has an unintentional weight loss of 20 pounds in the last 3 months. What should the nurse teach the patients family to prevent further loss of weight? 1. Provide liquid nutritional supplements with meals. 2. Add nonfat milk powder to scrambled eggs to add more protein. 3. Encourage the patient to resume smoking to increase the appetite. 4. There is nothing to change as weight loss is a normal part of aging.

2. Add nonfat milk powder to scrambled eggs to add more protein.

An older patient with moderate stage dementia frequently cannot remember which room he is assigned in a long-term care facility. Which nursing intervention would help this patient? 1. Reorient the patient when it happens again. 2. Establish landmarks at the patients bedside. 3. Investigate placing the patient in a private room. 4. Place the patient in restraints to limit ambulation.

2. Establish landmarks at the patients bedside.

An older patient without any major health problems is experiencing decreased strength and endurance while performing some activities. What should the nurse explain as the reason for the change in strength and endurance? 1. Depression 2. Decrease in lean muscle mass 3. Lowered absorption of vitamin D 4. Increase in cholecystokinin production

2. Decrease in lean muscle mass

An older patient with gastroesophageal reflux disease (GERD) is prescribed ranitidine (Zantac). What should the nurse instruct as the mechanism of action of this medication? 1. Neutralizes stomach acid 2. Decreases acid production in the stomach 3. Creates a coating that acts as a protective barrier 4. Increases motility in the esophagus and stomach

2. Decreases acid production in the stomach

An older resident is complaining of being constipated. Which action should the nurse take first when caring for this patient? 1. Assess the diet for adequacy of fiber and fluids. 2. Determine what the patient means by constipation. 3. Obtain an order for a laxative and an enema if needed. 4. Encourage the patient to increase fluid intake and activity.

2. Determine what the patient means by constipation.

The nurse has instructed an older patient on the modified My Plate and caloric intake. Which patent response indicates that instruction has been effective? 1. If I consume the lowest daily recommended value for all of the food groups, it will result in approximately 2,000 calories. 2. If I consume the lowest daily recommended value for all of the food groups, it will result in approximately 1,600 calories. 3. If I consume the lowest daily recommended value for all of the food groups, it will result in approximately 2,200 calories. 4. If I consume the lowest daily recommended value for all of the food groups, it will result in approximately 2,400 calories.

2. If I consume the lowest daily recommended value for all of the food groups, it will result in approximately 1,600 calories.

Which assessment finding should the nurse realize as being a cause for gastroesophageal reflux disease (GERD) occurring more commonly in older adults? 1. Increased amounts of saliva 2. Increased incidence of hiatal hernia 3. Tightening of the lower esophageal sphincter 4. The increase in peristalsis that occurs in the esophagus

2. Increased incidence of hiatal hernia

An older patient is receiving feedings through a permanent feeding tube. Which nursing intervention will decrease this patients risk of aspiration? 1. Administer formulas that contain fiber. 2. Keep the head of the bed elevated at a 30 to 45 degree angle. 3. The risk of aspiration no longer exists after a permanent feeding tube has been placed. 4. Flush the tube with water before and after each medication administered through the tube.

2. Keep the head of the bed elevated at a 30 to 45 degree angle.

An older patient receiving enteral feedings is experiencing abdominal cramps and liquid stools. Which ingredient in the patients tube feeding would cause these manifestations? 1. Maltose 2. Lactose 3. Fructose 4. Sucrose

2. Lactose

The nurse is caring for an older patient who is receiving phenytoin. Which nutritional issues is this patient at risk for developing? Standard Text: Select all that apply. 1. Altered swallowing 2. Reduced oral intake 3. Affected folate levels 4. Altered taste and smell 5. Affected vitamin D levels

2. Reduced oral intake 3. Affected folate levels 4. Altered taste and smell 5. Affected vitamin D levels

The nurse is planning a teaching session regarding gastrointestinal ulcers for the residents of an assisted living complex. Which concept should the nurse include in the presentation? 1. Gastric ulcers are more common than duodenal ulcers. 2. The first sign of a peptic ulcer may be serious gastrointestinal bleeding. 3. A colonoscopy is the test used to diagnose the presence of a gastric ulcer. 4. The individual having a peptic ulcer will most likely experience pain when hungry.

2. The first sign of a peptic ulcer may be serious gastrointestinal bleeding.

Which older patient is at greatest risk for vitamin D deficiency? 1. The patient with macrocytic anemia 2. The patient who does not drink milk 3. The patient who works outdoors daily and does not wear sunscreen 4. The patient who is taking isoniazid (INH) after a positive tuberculin skin test

2. The patient who does not drink milk

The nurse is caring for an older patient who is experiencing sleep deprivation. Which manifestation might the nurse assess in this patient? 1. Improved healing 2. Visual hallucinations 3. Fatigue occurring at night 4. Development of Alzheimers disease

2. Visual hallucinations

Which manifestations that an older patient is exhibiting should the nurse investigate as possibly being caused by peptic ulcer disease? Standard Text: Select all that apply. 1. Diarrhea 2. Clay-colored stools 3. Abdominal distention 4. Indigestion with bloating 5. Vague and diffuse abdominal pain

3. Abdominal distention 4. Indigestion with bloating 5. Vague and diffuse abdominal pain

An older patient with mild Alzheimers disease abruptly stops taking the prescribed medication donepezil (Aricept). On which area should the nurse focus when assessing this patient? 1. Reflexes 2. Rest and sleep 3. Cognitive function 4. Cardiovascular function

3.

The family of an older patient with Alzheimers disease does not want to discuss long-term care placement for at least a few years. How should the nurse respond to the family? 1. Long-term care placement is inevitable with this diagnosis. 2. It often takes a year for an individual with Alzheimers disease to be admitted. 3. Talking about it now gives you time to think about locations and make a decision. 4. By providing this information now, we will not need to address these concerns later.

3.

The nurse is planning interventions to reduce an older patients risk of dehydration. Which intervention would support the onset of dehydration in the patient? 1. Ensuring fresh water is available to the patient 2. Providing the prescribed diuretic with breakfast 3. Administering the prescribed diuretic with the evening meal 4. Offering a drink of water every time the patients room is entered

3. Administering the prescribed diuretic with the evening meal

Which statement about food insecurity would the nurse include in a presentation regarding nutritional issues in the older patient? 1. Food insecurity is when a person hoards food. 2. White older persons are at a higher risk for food insecurity than African Americans. 3. African American older persons are at a higher risk for food insecurity than Caucasian Americans. 4. Food insecurity is when a person is concerned that he or she is eating foods that might be harmful to his or her health.

3. African American older persons are at a higher risk for food insecurity than Caucasian Americans.

An older patient is telling the nurse about problems with sleeping. What does the nurse realize about sleep and the older adult? 1. The need for sleep decreases with age. 2. Disrupted sleep is not associated with depression. 3. A person should not awaken more than once during the night. 4. An older person does not have as much deep sleep as a younger person.

4. An older person does not have as much deep sleep as a younger person.

The nurse is preparing discharge instructions for an older patient with chronic pancreatitis. Which information should the nurse include in this teaching? 1. Stop smoking. 2. Limit fluid intake. 3. Do not drink any alcohol. 4. Avoid eating organ meats.

3. Do not drink any alcohol.

The nurse is concerned that an older patient is experiencing sleep apnea. What did the nurse assess in this patient? Standard Text: Select all that apply. 1. Jumpy legs 2. Sleeping with three pillows 3. Excessive daytime sleepiness 4. Excessive snoring upon inspiration 5. Complaints of choking when waking from sleep

3. Excessive daytime sleepiness 4. Excessive snoring upon inspiration 5. Complaints of choking when waking from sleep

What should the nurse instruct a caregiver to do to assist a cognitively impaired older patient to self-feed? 1. Offer the patient a variety of favorite foods. 2. Provide diversional stimuli, such as a television show, so the patient can eat without thinking about it. 3. Serve each food separately with the proper utensil and cue the patient to use the utensil to eat that particular food. 4. Place the food types in the same arrangement on the plate and relate the location to the face of a clock to assist the patient in locating the food on the plate.

3. Serve each food separately with the proper utensil and cue the patient to use the utensil to eat that particular food.

An older patient is experiencing diarrhea. Which assessment finding supports that the patients diarrhea is caused by Clostridium difficile? 1. The patient has a history of ulcerative colitis. 2. The patient has been taking prescribed steroids for several months. 3. The patient recently completed a course of antibiotics for pneumonia. 4. The patient rarely eats fresh fruits and vegetables and self-restricts fluid intake.

3. The patient recently completed a course of antibiotics for pneumonia.

An older patient, beginning antibiotic therapy for a leg wound, has a history of Clostridium difficile. What should the nurse instruct the patient to do to reduce the risk of this occurring? 1. Eat large amounts of fresh fruits and vegetables. 2. Restrict the amount of meat and calcium products. 3. Use acidophilus capsules while taking the antibiotic. 4. Decrease the amount of fluid taken while on the medication

3. Use acidophilus capsules while taking the antibiotic.

The nurse is discussing proper nutrition with older community members at a senior citizen center. What should the nurse teach as general guidelines for healthy older individuals? Standard Text: Select all that apply. 1. Calcium intake should be 1,000 mg for those over the age of 51 years. 2. Older individuals need to take supplements of vitamins A, C, E, and K. 3. Vitamin D intake should be 600 IU up to age 70 and 800 IU if older than 70. 4. Ingest at least 0.8 grams of protein for each kilogram of body weight each day 5. Fluid intake each day should be at least 13 cups for men and 9 cups for women.

3. Vitamin D intake should be 600 IU up to age 70 and 800 IU if older than 70. 4. Ingest at least 0.8 grams of protein for each kilogram of body weight each day 5. Fluid intake each day should be at least 13 cups for men and 9 cups for women.

An older patient begins to experience status epilepticus. Which action will the nurse take to help this patient? 1. Measure vital signs. 2. Orient the patient between seizures. 3. Prevent chilling with warmed bed linens. 4. Ensure an intravenous access line is available.

4.

An older patient with advanced Alzheimers disease is being treated for pneumonia. The daughter is not sure if resuscitation efforts should be a part of the plan of care. What information should the nurse provide to the daughter? 1. Resuscitation is often effective for older adults. 2. After resuscitation the patient will return to the same level of functioning. 3. As long as the resuscitation efforts are initiated quickly, the patient will survive. 4. Resuscitation for cardiac or respiratory arrest will have little probability of success.

4.

The nurse suspects an older patient is having a grand mal seizure. What did the nurse observe in the patient? 1. Rhythmic jerking of the muscles 2. Rigid extremities lasting for several minutes 3. Brief loss of attention similar to daydreaming 4. Rigid extremities followed by rhythmic flexion

4.

An older patient with diverticular disease is experiencing abdominal pain and fever. For which diagnostic test will the nurse most likely prepare this patient? 1. Colonoscopy 2. Barium enema 3. Upper GI endoscopy 4. CT scan of the abdomen

4. CT scan of the abdomen

An older patient with gastroesophageal reflux disease has complications caused by exposure of tissue to gastric acids. Which medication will this patient most likely be prescribed to promote healing? 1. Reglan 2. Zantac 3. Pepcid 4. Carafate

4. Carafate

What should the nurse teach an older patient about colorectal cancer? 1. The risk of colorectal cancer decreases with age. 2. Colorectal cancer can be detected by measuring carcinoembryonic antigen (CEA). 3. Colorectal cancer occurs less frequently in those with a history of ulcerative colitis. 4. Colorectal cancer has no symptoms but can be detected by fecal occult blood testing.

4. Colorectal cancer has no symptoms but can be detected by fecal occult blood testing.

How should the nurse instruct an older patient with gastroesophageal reflux disease (GERD) about heartburn? 1. It improves when lying flat or bending over. 2. It is unaffected by the size of meals eaten or the types of food. 3. It will not put the older patient at increased risk for esophageal cancer. 4. It may cause severe chest pain that causes the patient to fear a heart attack.

4. It may cause severe chest pain that causes the patient to fear a heart attack.

An older patient with a history of constipation has been directed to use an over-the-counter medication to help manage symptoms of gastroesophageal reflux by buffering the gastric pH. Which medication would be the best for the patient to use? 1. Pepcid 2. Maalox 3. Mylanta 4. Milk of Magnesia

4. Milk of Magnesia

An older patient is prescribed diet supplementation to combat unintentional weight loss. How should the nurse provide these supplements to the patient? 1. Serve at room temperature. 2. Provide with the next meal. 3. Provide separate from medications. 4. Provide more than an hour before the next meal.

4. Provide more than an hour before the next meal.

Which older patient would the nurse identify as being at the highest risk of dehydration from receiving nutrition through a feeding tube? 1. Receiving bolus feedings 2. Receiving 50 ml free water at 4-hour intervals 3. Receiving feedings through a jejunostomy tube 4. Receiving feedings with a formula that is 1.5 calories per ml

4. Receiving feedings with a formula that is 1.5 calories per ml

An older patient is diagnosed with a large peptic ulcer. Which information will the nurse use from the patients history to help identify the cause for this ulcer? Standard Text: Select all that apply. 1. Allergy to penicillin 2. History of cataract surgery several months ago 3. Taking prescribed medication for hypertension 4. Taking ibuprofen (Motrin) for chronic bursitis pain 5. Prescribed warfarin (Coumadin) for chronic atrial fibrillation

4. Taking ibuprofen (Motrin) for chronic bursitis pain 5. Prescribed warfarin (Coumadin) for chronic atrial fibrillation

A care conference is being held with the family of an older patient in the late stages of Alzheimers disease. The family wants to know if a feeding tube should be placed. How should this question be answered? 1. Tube feeding will aid the patient to gain weight. 2. Tube feeding is associated with a reduced risk of aspiration. 3. Tube feeding reduces the discomfort associated with dehydration. 4. The absence of tube feeding promotes dehydration which reduces pain sensitivity.

4. The absence of tube feeding promotes dehydration which reduces pain sensitivity.

A resident in the nursing home is diagnosed with undernutrition and is unable to take in adequate food despite efforts by the multidisciplinary team and family members. Prior to insertion of a permanent feeding tube, which issue needs to be considered? 1. Equipment, care, and time needed to administer the feedings 2. The extent of the surgical intervention, cost and insurance coverage 3. The patients nutritional needs and tolerance of the formula feedings 4. The patients advanced directive and evaluation of risks, benefits, and ethical considerations

4. The patients advanced directive and evaluation of risks, benefits, and ethical considerations

The nurse instructs a family member on how to feed an older patient. Which observation indicates that the family member needs additional instruction? 1. Checks to make sure the patients dentures are in place 2. Makes sure that each bite is swallowed before providing the next bite 3. Reminds the patient to chew the food after being placed in the patients mouth 4. Tries to insert a utensil in the patients mouth and the patient bites down tightly

4. Tries to insert a utensil in the patients mouth and the patient bites down tightly

4. Which of the following nursing interventions should be the priority for a nurse working in a retirement community? A) Using restraints to keep nursing home residents from getting out of chairs unattended B) Establishing a fall-prevention program for residents at risk C) Using cordless phones or emergency call systems for residents in assisted living D) Using a monitoring device for people who live alone in their own home

Ans: B

1. A nurse is teaching health interventions to an older adult with osteoarthritis. Which of these statements indicates that the individual needs additional teaching? A) "I will avoid high-impact exercises." B) "I will get adequate intake of calcium and vitamin D." C) "I will try to limit my use of walkers and assistive devices." D) "I will lose weight if it turns out that I need to."

Ans: C

15. Which of the following hospitalized older adults is at greatest risk for in-hospital hip fractures from a fall? A) A 79-year-old client B) A client receiving numerous cardiac medications C) A client with a history of hip fractures from a fall D) A client with new-onset dementia

Ans: C

2. A 69-year-old has recently been diagnosed with mild cognitive impairment and has asked the nurse to help her remember things better. Which of the following nursing diagnoses is appropriate for this older adult? A) Knowledge deficit B) Altered thought processes C) Health-seeking behaviors D) Altered health maintenance

Ans: C

2. A new nursing assistant asks the nurse how best to approach a hearingimpaired older adult. Which of the following approaches should the nurse recommend? A) Raise the volume of your voice. B) Leave the radio on to calm the older adult. C) Lower the tone of your voice. D) Use exaggerated lip movements.

Ans: C

14. A rural community nurse sets up a monthly disease screening service. Local news coverage chooses not to cover this event. Which of the following justifications best supports the screenings? A) Health promotion is essential for older adults because they have complex health care needs. B) Older adults are less responsive to health promotion interventions. C) Older adults as a group receive fewer prevention and screening services than other populations. D) Preventive services are less effective after the onset of chronic illness.

Ans: A

10. A hospice nurse admits an older adult after the metastasis of malignant melanoma. Which of the following health problems should the nurse anticipate? A) Necrosis of extremities B) Hemoptysis C) Hyperglycemia D) Dyspnea

Ans D

1. A 76-year-old Hispanic woman has been admitted to the hospital. There are no advance directives in the chart. Which of the following questions will best prepare the nurse to begin a dialogue with this client about advance directives? A) "With whom do you talk to about your health care decisions?" B) "I see that you have no advanced directives on your chart, could you tell me about that?" C) "Is there someone we should call to join us while we discuss your care?" D) "Tell me about your living arrangements; do you live alone or with others?"

Ans: A

1. A nurse develops a plan of care for an older adult recently diagnosed with Lewy body dementia. Which functional consequence would be most important to monitor in this older adult? A) Development of visual hallucinations B) A visual acuity score of 20/30 C) Improved visual acuity after medications for dementia D) Growth of cataracts

Ans: A

1. A nurse teaches an older adult about changes to nutritional requirements. Which of the following meal choices would give evidence that the older adult understands the teaching? A) Baked chicken, carrots, and angel food cake B) Green salad, mashed potatoes, and an oatmeal cookie C) Vegetable beef soup, crackers, and Jell-O D) Baked pork chop, green beans, and sherbet

Ans: A

1. An 80-year-old is seen in the emergency department for a fall. The client has bruises on the upper arms and appears depressed. The client is accompanied by a grandchild, who is unkempt, glassy-eyed, and whose breath smells of alcohol. Which of the following should be a priority with the nurse? A) Assess whether the older adult is safe in the home environment. B) Determine whether legal interventions are appropriate. C) Assess the client's degree of frailty and chronic health problems. D) Determine the mental capacity of the older adult.

Ans: A

10. A nurse admits a 90-year-old client to the hospital with a diagnosis of failure to thrive. Which of the following laboratory data should the nurse expect? A) Low albumin and red blood cells B) Elevated white blood cells (WBCs) and low potassium C) Low platelets and low prothrombin time (PT) D) Elevated calcium and magnesium

Ans: A

10. A resident of a nursing home has experienced a progressive loss of vision over the past several months as a consequence of diabetes. How should the nurse accommodate the resident's loss of visual acuity? A) Provide the resident with brightly colored grooming utensils. B) Replace the resident's tube television with a flat-screen TV. C) Remove books from the resident's room to avoid reminding her of her vision loss. D)Have the walls in the resident's room painted a neutral color that matches the color of the flooring

Ans: A

11. A nurse assesses an older adult's color perception. Which of the following colors should the nurse expect the client to have the most difficulty visualizing? A) Blue and violet hues B) Yellow tones C) White and off-white D) Tan and brown wavelengths

Ans: A

11. An emergency room nurse cares for the family of a 70-year-old African American woman who died unexpectedly. In the waiting room, upon hearing of the death, two family members kneel to the floor moaning and do not respond. Which intervention by the nurse is most appropriate at this time? A) Allow the family to grieve in this manner directing others away. B) Assess these family members' vital signs and neurologic status. C) Bring these family members to the body of their loved one. D) Call an emergency response team to care for these family members.

Ans: A

12. An older adult with heart failure and mild dementia states the intent to refuse low sodium diet and diuretics, stating: "It's important to me to live free, without restrictions on what I eat." The family is supportive. Which action, by the nurse, should be done first? A) Assure that the client understands the consequences of this decision. B) Discuss this decision with the older adult's family to plan for the future. C) Document the client's wishes in the plan of care. D) Notify the primary health care provider of the client's wishes.

Ans: A

13. A nurse teaches an older adult about risks related to ototoxic medications. Which of the following medications should the adult minimize or avoid? A) Nonsteroidal anti-inflammatory agents B) Osmotic stool softeners C) Over-the-counter sleep aids D) Penicillin-type antibiotics

Ans: A

14. A nurse at a rehabilitation unit assesses an 86-year-old woman with a BMI of 30 and a history of heart failure, whose oral intake is declining. Which of the following risk factors is related to this older adult's decline in appetite? A) Diuretics B) Exercise C) Female gender D) Obesity

Ans: A

14. A nurse plans care for a frail older adult in long-term care. Which of the following interventions should be included in the plan of care to reduce the risk of respiratory infections? A) Oral care B) Oxygen administration C) Pulmonary function testing D) Tracheal suctioning

Ans: A

8. A 78-year-old client has been brought to the emergency department from home with a sudden change in mental status accompanied by significant weakness. For which condition should the health care providers assess? A) Alzheimer disease B) Lung cancer C) Pneumonia D) Tuberculosis

Ans: C

15. A nurse plans discharge instructions of a 78-year-old black client with newly diagnosed dilated heart failure. The client states, "Will I be going home on hospice now that my heart is failing?" Which of the following responses is most appropriate by the nurse? A) "Heart failure is a chronic condition that can be controlled with medication." B) "No, but you will have palliative care." C) "You must go to cardiac rehabilitation." D) "You will need to take medications for the rest of your life."

Ans: A

15. An older adult expresses frustration about limitations of aging. Which of the following statements by the nurse promotes wellness? A) "Do you have some words of wisdom to share about that valuable experience?" B) "How does living in these conditions compare to your youth?" C) "Have you met any of your neighbors, they seem like nice people?" D) "What you are saying is that you are frustrated with how they are not listening to you?"

Ans: A

2. A 65-year-old client with a long-standing history of chronic obstructive pulmonary disease (COPD) was placed recently on Coumadin after experiencing atrial fibrillation. Upon discharge from the hospital, which of the following statements by the client indicates a need for further teaching? A) "I will continue to use smokeless tobacco since it's a lot better than smoking." B)"I will avoid using over-the-counter antihistamines since they can dry my mucosal secretions." C)"I will watch my intake of dark green leafy vegetables since they may impact the effects of Coumadin." D)"I will not take any herbal preparations without my health care provider's knowledge."

Ans: A

2. During assessment of an older adult, the nurse discovers that the individual has been reluctant to divulge recent losses in activities of daily living to his primary care provider. Which of the following factors has been demonstrated to contribute to such reluctance? A) The older adult may fear a loss of independence if problems are disclosed. B) The older adult may realize that age-related changes are normally not treatable. C) The older adult may be experiencing cognitive deficits that influence decision making. D) The older adult may recognize that health care systems are not able to address psychosocial problems.

Ans: A

3. A 99-year-old resident has fallen. Which of the following functional consequences of this fall most strongly impacts the plan of care? A) A 99-year-old is at much higher risk of a fracture from a fall than a younger person. B) A 99-year-old is more likely to have limited range of motion, impacting performance of some activities of daily living (ADLs). C) A 99-year-old who has fallen is unlikely to develop fear of falls. D) A 99-year-old will have diminished muscle strength related to muscle mass loss.

Ans: A

3. A neighbor notices an 81-year-old getting water from someone's outside faucet. The neighbor notices that this person's ankles are very swollen and there is an open wound on her left leg. The older adult says, "I stopped taking my pills because the water department turned off my water and I can't use the bathroom. My daughter did not pay the water bill, and she never has time to take me to the doctor so my legs can be checked." The neighbor calls adult protective services. Which of the following interventions is the priority when the nurse visits for an evaluation and does not find any immediate danger? A) The competency of the older adult in making decisions needs to be determined. B) The daughter needs to be picked up by the police on a neglect charge. C) The older adult needs to be involuntarily committed to a long-term care facility. D) An involuntary legal intervention needs to be initiated immediately.

Ans: A

3. A nurse assesses the urinary elimination of older adults. Which of the following actions by the nurse is most appropriate? A) Work to identify terms that the older adult comprehends. B) Wait until the person initiates a discussion of this embarrassing topic. C) Give the interview questions to the client in writing. D) Ask the older adult to keep a urination diary

Ans: A

4. A nurse cares for an older adult who is nonresponsive and surrounded by family. Which of the following statements by the nurse is most appropriate? A) "I am here for you, how can I be helpful?" B) "I know just how you feel, my mother died last month." C) "I'll leave you alone so that you can grieve in private." D) "She was a really nice lady; she did a good job raising you."

Ans: A

4. A nurse completes assessment of an older adult. Which of the following physical assessment findings is within normal limits? A) Kyphosis and increased anteroposterior diameter of the chest B) Increased intensity of lung sounds C) Decreased resonance on percussion D) Decreased adventitious sounds in lower lungs

Ans: A

4. Which of the following interventions most closely aligns with the practices of health promotion? A) Leading a flexibility and mobility class among older adults B) Providing presurgical teaching to an older adult prior to hip replacement C) Administering an anti-inflammatory and analgesics to an older adult with osteoarthritis D) Teaching an older adult how to administer her inhaled bronchodilators independently

Ans: A

5. Which of the following older adults is most at risk to develop osteoporosis? A) A 65-year-old white woman with chronic obstructive pulmonary disease who takes corticosteroids B) A 65-year-old white man with rheumatoid arthritis C) A 70-year-old African American man with a seizure disorder D) A 68-year-old Hispanic woman who recently had a partial hysterectomy

Ans: A

5. Which of the following statements by the new nurse best conveys an understanding of diabetes in older adults? A) "A combination of lifestyle factors and age-related changes contributes to high rates of diabetes among older adults." B) "Development of diabetes later in life is considered a normal, age-related change." C) "Health care providers should avoid drawing conclusions about diabetes risk based on ethnicity." D) "The diagnosis of diabetes in older adults is complicated by subtle signs and symptoms."

Ans: A

6. A 76-year-old adult expresses frustration to the nurse regarding hearing loss despite a lifetime of being conscientious about avoiding known causes of hearing damage. Which of the following age-related changes may result in hearing loss? A) Degeneration of the inner ear structures B) Decreased viscosity and quantity of cerumen C) Plaque formation and occlusion of the Eustachian tubes D) Hypertrophy of the external ear structures

Ans: A

6. A nurse in a long-term care facility is aware of the effects of age-related changes to the respiratory system. Which of the following functional consequences most likely results from age-related changes? A) Snoring and mouth breathing B) A persistent, dry cough C) Increased sensitivity to environmental allergens D) Hemoptysis on exertion

Ans: A

7. A 66-year-old has been conscientious about health as an adult and is disappointed at having been recently diagnosed with type 2 diabetes. The client had been unwilling to discuss this new diagnosis for the past several weeks but has now begun asking the nurse questions about this disease. Which of the following nursing diagnoses is most appropriate for this client? A) Readiness for enhanced knowledge B) Readiness for enhanced self-care C) Readiness for enhanced power D) Readiness for enhanced comfort

Ans: A

7. A nurse assesses a 70-year-old man who has high blood pressure and chronic obstructive pulmonary disease (COPD). He has been prescribed nicardipine and ipratropium inhaler. This medication combines a calcium-channel blocker and an anticholinergic. For which of the following urinary effects should the nurse teach the client to monitor? A) Nocturia B) Urinary tract infection (UTI) C) Urge incontinence D) Hematuria

Ans: A

7. A nurse in an intensive care unit prepares to perform postmortem care on an older Jewish client. Family members are at his bedside. Which of the following actions by the nurse is appropriate? A) Allow the family to remain with the client. B) Liaise with the hospital chaplain to visit the family in the chapel. C) Address the man's oldest son when discussing the client's cares. D) Determine which family member(s) will be staying at the bedside during the cares

Ans: A

8. A nurse assesses risk factors for vision loss in a 71-year-old client. Which question should the nurse include in this assessment? A) "Do you have high blood pressure or diabetes?" B) "Did your parents wear glasses or have cataracts?" C) "How much red meat do you usually eat?" D) "Do you have high cholesterol?"

Ans: A

8. A nurse plans activities each month at an assisted living facility. Which of the following activities is most cognitively stimulating? A) Book discussions B) Movie night C) Exercise D) Reminiscence therapy

Ans: A

8. An 82-year-old client walked 2 miles last week to enjoy the spring weather. The client says since that time, "I haven't been doing very much, I'm afraid it will hurt." Which action by the nurse is most appropriate? A) Discuss moderation in activity, encouraging continued movement. B) Obtain a cane for use to improve balance, and reduce the client's fears. C) Encourage the client to walk the 2 miles every day. D) Have the client take ibuprofen (Motrin IB) every morning.

Ans: A

9. A 70-year-old woman has expressed interest in preventing osteoporosis as a result of the high prevalence of the disease in her peer group. What dietary measures should the nurse recommend? A) High intake of salmon and fortified cereals B) A high-protein, low-carbohydrate diet C) High intake of organic fruits and vegetables D) Vitamin C supplements and a high-potassium diet

Ans: A

9. A wound care nurse is assessing a 76-year-old client. The client has intimated to the nurse that her son sometimes "flies off the handle and gets rough with me." Which response made by the nurse is the best response? A) "When you say 'gets rough,' what does that look like?" B) "What do you think usually provokes this to happens?" C) "I'm going to have to phone adult protective services right now." D) "Why do you think that there is that response with anger or frustration?"

Ans: A

1. A nurse develops a plan of care for a family with nursing diagnosis of Caregiver Role Strain related to urinary incontinence. Which of the following interventions is the highest priority? A) Administer diphenhydramine at bedtime. B) Assist the client to the bathroom prior to bedtime. C) Limit the fluid intake of the client to 1000 mL each day. D) Monitor bowels for diarrhea and constipation

Ans: B

1. A nurse is responsible for assessing an older adult in an acute care setting. Which of the following statements most accurately captures the complexity involved in assessing the older adult? A) Older adults manifest fewer symptoms of illness than do younger clients. B) Signs and symptoms of illness are often obscure and less predictable among older adults. C) Care must be taken to avoid assessing normal, age-related changes. D) Older adults experience fewer acute health problems but more chronic illnesses than do younger clients

Ans: B

1. Which of the following statements best captures the typical character of health problems in the lives of older adults? A) Older adults' lives are dominated by the increasing number of acute health problems due to age-related changes. B) Most older adults experience an interplay between a number of chronic conditions and occasional acute health problems. C) Older adults can expect a relatively consistent decline in their health over time as a result of acute health problems. D) Chronic conditions account for the normal downward direction of an older adult's health status

Ans: B

10. A nurse interviews a client who is 82 years old and has several chronic conditions, including type 2 diabetes and heart failure. The client expresses feeling of more satisfaction with life now than when younger. Which phenomenon is the client expressing? A) Metamemory B) The paradox of well-being C) Crystallized intelligence D) Neuroplasticity

Ans: B

10. Assessment of an older adult's ADLs addresses parameters such as mobility, dressing, and elimination. In addition to these, which of the following categories should the functional assessment also include? A) Pain B) Mental status C) Previous medical history D) Integumentary assessment

Ans: B

10. Following a prolonged hospital stay due to an exacerbation of congestive heart failure, an older adult woman has returned to the nursing home where she normally resides. The woman became incontinent of urine during her time in the hospital, a problem that nursing staff wish to now resolve. What action should her caregivers take in performing continence training? A) Limit the woman's fluid intake to 750 mL daily, primarily before suppertime. B) Assist the woman with toileting at timed intervals throughout the day. C) Teach the woman about the functional and psychosocial benefits of restoring continence. D) Perform intermittent catheterization before each meal and before bedtime

Ans: B

11. A gerontological nurse presents at a local conference regarding recent findings in age-related hearing loss. Which age-related changes indicated in hearing loss and speech perception should the nurse include? A) Changes to the external auditory canal and pinna B) Degenerative changes to the auditory brainstem C) Growth of longer and thicker hair in the ear canal D) Thickening of the tympanic membrane

Ans: B

11. A nurse assesses an older adult 24 hours after a retropubic suspension surgical procedure. The client is confused, exhibits muscle twitching, and states she is nauseated. Which laboratory data should the nurse assess? A) Blood urea nitrogen B) Sodium C) Hemoglobin D) White blood cell count

Ans: B

11. A nurse assesses older adults at a senior center. One older adult, age 78, has a body mass index (BMI) of 15. Which response by the nurse is appropriate? A) "You are too skinny." B) "Have you been losing weight?" C) "Have you tried to lose this extra weight?" D) "Congratulations your BMI is great."

Ans: B

11. A nurse discusses common illnesses at the local health fair. The older adult asks, "Why do all my friends seem to get pneumonia?We never did when we were younger." Which of the following interventions should the nurse include in the teaching? A) Examinations by health care provider B) Hand hygiene C) Jogging/running D) Yearly pneumovax

Ans: B

12. A graduate nurse expresses an interest in focusing future study to healthy aging. Toward which of the following fields should the experienced nurse steer the graduate nurse? A) Palliative care B) Gerontological nursing C) Nursing home administration D) Social work

Ans: B

12. A nurse cares for a 92-year-old woman with urinary incontinence. Which of the following agerelated changes is the rationale behind assisting client to the bathroom every 2 hours? A) Decreased estrogen levels B) Degenerative changes in the cerebral cortex C) Demyelination of parasympathetic nerves D) Diminished thirst perception

Ans: B

12. A nurse manager of an intensive care unit develops plans to improve end-of-life care for clients in the unit. Which of the following actions is the priority? A) Create a script for nurses to use when discussing hospice and palliative care. B) Guide staff to improve communication with families about end-of-life decision making. C) Increase communication between professionals about end-oflife decision making. D) Survey clients and families about their end-of-life needs.

Ans: B

12. The 64-year-old client who went rock climbing last week and snowboarding this week is at risk for broken bones. Which functional consequence of aging most strongly increases this risk? A) A strong musculoskeletal system helps to protect bones. B) Reduced osteoblastic production of bone matrix C) The long bones have decreased blood flow with aging. D) Weight-bearing activities increase calcium uptake into bones

Ans: B

13. A nurse assesses a 79-year-old adult noting the presence of a white ring around the iris bilaterally. What is the correct term for this? A) Glaucoma B) Arcus senilis C) Arthritis D) Presbyopia

Ans: B

13. A nurse cares for an older adult in a residential care program. The client has multiple chronic conditions. The client has developed dyspnea and has lost 105 lb of body weight. Which of the following statements by the nurse is most appropriate? A) "Have you ever heard of palliative care?" B) "I want to talk to you about switching our focus from cure to care." C) "We don't think that there is anything we can change to make you better." D) "Your breathing problems concern me."

Ans: B

13. A nurse determines that a client does not remember current events and has difficulty using technology. The nurse should consider that the client may have difficulty with which of the following? A) Participating in reminiscence group B) Digitally recording blood glucose monitor C) Remembering to weigh daily D) Understanding when to notify health care provider

Ans: B

14. A home health nurse visits a client who has a history of alcoholism and dementia. The client's words are incoherent, and the client's clothes are filthy. The client is unsteady and leaning to the right, and the room in the rooming house is in disarray. The only word that the nurse can clearly identify is "no." Which action by the nurse is most appropriate at this time? A) Ask the neighbors what has been happening. B) Call emergency services for transport to a hospital. C) Leave and return later. D) Search the room for empty bottles.

Ans: B

14. The nurse presents at a conference regarding aging and mobility. Which age-related changes should the nurse include? A) Bones decrease resorption due to decreased parathyroid hormone B) Diminished positioning sensations in the lower extremities C) Outgrowth of collagen and elastin cells D) The number of skeletal muscle fibers increases

Ans: B

15. A nurse helps older adults increase healthy behaviors by restructuring. Which of the following communication techniques should the nurse use to help the older adult overcome barriers? A) "Could you walk with your friend for a half-hour after you both come back from the lunch program at the senior center?" B) "I know it's hard to get outside in the winter, so let's try to identify some ways of getting more exercise indoors during your usual activities." C) "Let's talk about the benefits of exercise. When you walk regularly you reduce constipation and muscle pain." D) "Your activity has been decreasing during the last 3 years and it is at the point that you are at an increased risk for heart disease."

Ans: B

2. A nurse cares for an 87-year-old client from India who has noted Hinduism as religious preference on admission records. This client is transferred to the hospice unit. Which of the following actions by the nurse best shows caring? A) Ask the family to tell you about Hinduism. B) Assess the client's spiritual needs. C) Notify the family's pastor. D) Pray with the client and family.

Ans: B

3. A 70-year-old client smoked for 30 years and has a history of COPD. The spouse assists with cooking, cleaning, and transportation. The spouse has become ill, and they now receive assistance from a home health nurse. Which of the following interventions should be the priority? A) Assisting the clients to perform instrumental activities of daily living (IADLs) B) Determining a plan for providing meals C) Setting up medications for the clients D) Smoking cessation plan

Ans: B

3. A 70-year-old client with urosepsis is admitted to a nursing unit. The labs include elevated sodium, blood urea nitrogen, hematocrit, and albumin. Which of the following nursing diagnoses is priority for this client? A) Constipation B) Fluid volume deficit C) Imbalanced nutrition: less than body requirements D) Impaired tissue perfusion

Ans: B

3. A nurse conducts a functional assessment of a client who has moved to the assisted living facility. Which of the following statements best describes this functional assessment? A) Information on the client's medical diagnoses and health problems. B) Client's ability to perform self-care tasks with a focus on rehabilitation. C) Assessment of the client's activities of daily living (ADLs). D) Prioritization of the client's ability to perform roles in relationships and in society.

Ans: B

3. A nurse explores resources available to assist a client. Which of the following older adults meets the eligibility requirements for hospice care? A) A client who is immobilized and unable to afford specialized nursing care B) A client who has experienced a stroke and been given 3 months to live C) A client with cancer who is living with uncontrolled persistent pain D) A client with acquired immunodeficiency syndrome who lacks family support to provide needed care

Ans: B

3. A nurse is teaching new graduates about the nature of palliative care on her unit. Which of the following statements by a new nurse indicates a need for further teaching? A) "I can see how important it is for us to educate patients and their families and friends on the unit." B) "It certainly requires a change in thinking to understand why we don't provide any medical interventions for patients." C) "I can see how comfort and psychosocial well-being take precedence over physical functioning." D) "Purpose in life and quality of life seem to be the overarching goals of palliative care."

Ans: B

3. A nurse is teaching nursing students about the importance of health promotion among older adults. Which of the following statements by a student indicates a need for further teaching? A) "Teaching older adults how to live with diabetes would be a useful health promotion initiative." B) "I can see why we would want to promote timely discharge back to the community following hospitalization." C) "I think that attending to spiritual growth could likely be a part of health promotion." D) "If we could promote healthy, simple diets, then some diseases could likely be prevented."

Ans: B

4. A nurse assesses a 61-year-old adult who reveals that he can't process as quickly as when younger, and that "all these people talk about multi-tasking, but I can't do that!" Which of the following responses by the nurse is appropriate? A) "Have you had any other symptoms of cognitive impairment?" B)"Slower processing of information is an age-related change, and there are things you can do to help with this." C) "The declines in cognitive skills usually begin around the age or 60." D) "You shouldn't expect to see a decline the cognitive functions that you use all the time."

Ans: B

4. A nurse notes that a client who has heart failure could hear well during the last home visit, and is having difficulty hearing today. Which of the following laboratory findings is most likely associated with impaired hearing? A) Albumin of 4.1 B) Creatinine of 4.2 C) Potassium of 4.3 D) Sodium of 144

Ans: B

4. A nurse plans the diet for an older adult with congestive heart failure. Which of these nursing interventions would be most successful to encourage optimal nutrition? A) Encourage calorie supplements. B) Provide 55% of calories from complex carbohydrates. C) Teach older adults to sit upright for 2 hours after a meal. D) Use moderate to large amounts of flavor enhancers

Ans: B

4. A nurse teaches an older adult man to perform pelvic floor muscle exercises (PFME)? Which of the following should be included in a nurse's instructions? A) Interrupt the flow of urine several times each time you urinate. B) Identify the correct muscle by making the base of your penis move up and down. C)Contract your legs and buttocks while contracting the pubococcygeal muscle. D) Perform the exercise while standing over the toilet

Ans: B

5. A 30-year-old grandchild lives with and provides care for the 75-year-old grandparent. The grandparent has congestive heart failure, hypothyroidism, and chronic pain from a compression fracture and osteoporosis. The grandchild supervises the older adult's medications. The home health nurse notes that the older adult has extra diuretic pills and that the pain medications for a month have been used and cannot be refilled for 2 more weeks. The older adult tells the nurse: "Those pain pills don't work, my back is always hurting." The nurse notes that the older adult's ankles are very swollen. Which of the following things should the nurse do first? A) Call adult protective services and ask for an immediate evaluation. B) Assess the grandchild's understanding of her grandmother's needs. C) Take the grandmother to the emergency department immediately. D) Tell the older adult that her grandchild is probably taking her pain medications.

Ans: B

5. A nurse in a Medicare- and Medicaid-funded nursing home performs assessments and develops care plans. Which of these statements is true of the functional assessments the nurse is likely to perform? A) The nurse will address core ADLs but not more complex IADLs. B) The nurse will identify changes in the older adult's function over time. C) The nurse will utilize various functional assessment models. D) The main goal of functional assessments will be to ensure older adult safety.

Ans: B

5. A nurse knows teaching has been effective when the client states the following: A)"If my sensitivity to glare decreases and my contrast sensitivity increases, I will be evaluated for cataracts." B)"I wear sunglasses and a wide-brimmed hat when I am in the sun to protect my eyes and prevent the development of cataracts." C)"Having Alzheimer disease increases one's risk of developing macular degeneration." D)"If I take ototoxic medications, this will increase my risk for developing cataracts."

Ans: B

5. A nurse manager of a nursing care facility reviews potential health and safety interventions and outcomes with the nursing staff. Which of the following interventions places emphasis on quality of life with the best possible health and safety outcomes? A) Telling the resident about his or her schedule for the day B) Allowing the resident with a history of falls to walk with the help of an assistant C) Discouraging the resident who states they are tired from participating in activities D) Advising the resident not to attend church because the resident is not Catholic

Ans: B

5. An 85-year-old woman who lives alone says to the nurse, "There is nothing I can do about my hearing. I am 85 years old, and I am not really interested in listening to television programs anymore." Which of the following would be the nurse's best response? A)"You are lucky you still live alone at 85, and I understand why you don't care about the programs on television." B)"Have you talked with your health care provider about a hearing evaluation? This would determine the problem and possible solutions to it." C)"I know a hearing aid dealer who offers free testing. Have you thought about trying a hearing aid?" D)"Did you know that there are closedcaption television sets that would allow you to enjoy some shows?"

Ans: B

5. An older adult client on a palliative care unit divulges to his nurse that he fears being in pain during the dying process. How can the nurse best respond to the client's admission? A) "It sounds like you're understandably anxious about this. How can I help you to relax?" B) "A lot of clients do have pain at the end, and we will do all we can to control it for you." C) "We will do all we can to address it, and you'll be able to have a peaceful passing." D) "Pain during the dying process actually is not nearly as common as many people believe."

Ans: B

5. Which of the following points should the nurse emphasize when educating older adults about memory and cognition? A) Long-term memory loss is normal. B) Using calendars, notes, and imagery can help enhance memory. C) Drinking caffeinated beverages for mental stimulation is a good idea. D) Having a diminished capacity for learning is an inevitable part of growing older.

Ans: B

6. A 78-year-old was diagnosed with colorectal cancer 18 months ago and underwent a round of chemotherapy. The most recent computed tomographic scan, however, reveals that the cancer has metastasized to the lungs and liver. The older adult states, "I feel quite well and do not wish to undergo another round of chemotherapy. " The client's children are adamantly opposed to their parent's decision to forgo treatment and have appealed to the nurse. Which factor is the priority consideration for the nurse to determine the best course of action? A) The client's prognosis B) The client's autonomy C) The family's wishes D) The client's treatment options

Ans: B

6. An 81-year-old client was diagnosed with colon cancer several months ago. Both the patient and the family have been made aware of the potential for metastasis and the poor prognosis associated with this disease. At what point in the client's disease trajectory should the principles of palliative care be implemented? A) After chemotherapy and radiation therapy have been proven unsuccessful B) Early in the course of the client's cancer and before symptoms become unmanageable C) Soon after the client has made a decision to change her code status to "do not resuscitate" D) When the client's care team determines that there is less than 2 months to live

Ans: B

7. A nurse who provides care in a clinic comes into contact with numerous older adults, many of whom have bruises of various sizes and stages on their body. What pattern of bruising is most suggestive of possible abuse? A) Significant bruising on the shin region of a client's leg B) Bruising on both ears and both sides of the neck C) Bruising on the back of a client's hands D) Bruising on both of a client's elbows

Ans: B

7. After a scheduled trip to her optometrist, a 70-year-old has been told that the pressure in her eye is high and she needs to be monitored and treated to prevent damage to the optic nerve. What is this person's diagnosis? A) Cataracts B) Glaucoma C) AMD D) Presbyopia

Ans: B

7. Despite the fact that older adults are proportionately the highest users of health care services, many nurses harbor misconceptions and deficits in practice related to gerontological nursing. What is the most likely solution to this problem? A) A shift from the treatment of older adults in institutional settings to home care B) Increased nursing education and clinical experience specific to working with older adults C) A focus on early discharge planning for older adults in hospital settings D) Increased use of aggressive pharmacologic interventions in the treatment of acute illnesses in older adults

Ans: B

7. The children of a resident of a nursing home have approached the nurse because they believe their parent is being manipulated by a person who also lives in the facility. Their parent has a diagnosis of early-stage Alzheimer disease and various comorbidities that affect mobility and function. How should the care team appraise the parent's decision-making capacity? A) Her decision-making ability is nullified by the presence of a dementia. B) Her decision-making capacity should be determined according to objective criteria. C) She should be asked to demonstrate sound decision making in minor matter before being allowed to make more important decisions. D) A surrogate should be appointed to make her decisions because she has been diagnosed with Alzheimer disease.

Ans: B

8. An 89-year-old woman has developed urinary incontinence. The woman states, "When I have to go, I go. I can't make it to the bathroom before it leaks out." For which type of incontinence should the nurse develop a plan of care? A) Functional incontinence B) Urge incontinence C) Mixed incontinence D) Stress incontinence

Ans: B

9. A community health nurse is to create initiatives to foster the health of older adults in the community. Which of the following health promotion activities has the greatest potential to promote the respiratory health of older adult participants? A) A lung cancer screening program B) A smoking cessation program C) A tuberculosis screening program D) A bronchitis immunization program

Ans: B

9. A nurse discusses driving with an older adult who continues to drive, but is probably unsafe on the road. Which statement made by the nurse is most appropriate? A) I am calling your child to take your keys. B) I am concerned about your safety, as well as the safety of others. C) We just don't want you to crash when you drive across the state. D) You shouldn't drive anymore.

Ans: B

9. As part of a comprehensive physical assessment of an older adult client, a nurse is performing an otoscopic examination. Which of the following assessment findings most clearly warrants further assessment and possible intervention? A) There is a small amount of cerumen visible in the ear canal. B) The epithelial lining is bright red. C) The tympanic membrane is intact. D) The tympanic membrane is a pearl-gray color

Ans: B

1. A 62-year-old who has worked on an assembly line since he was 24 years old began taking aspirin for arthritis 6 months ago. The client presents to the nurse with hearing problems and ringing in the ears. Which of the following problems should the nurse suspect? A) Tinnitus B) Vertigo C) Ototoxicity D) Impacted cerumen

Ans: C

1. A nurse assesses an older adult's overall respiratory function. Which of the following interview questions would be most appropriate? A) "Would you be interested in finding out more about environmental smoke?" B) "Did either of your parents experience lung diseases?" C)"Have you ever worked in a job where you were exposed to dust, fumes, smoke, or other pollutants?" D) "What do you do to actively maintain your respiratory health?"

Ans: C

1. A nurse is explaining to a new colleague the similarities and differences between gerontology and geriatrics. Which of the following situations would most likely be addressed by a geriatrician rather than a gerontologist? A) Teaching older adults techniques to identify and deal with age-related changes B) Organizing and leading exercise classes to facilitate mobility C) Identifying and treating a client's vascular dementia D) Rearranging an older adult's apartment to minimize the risk of falls

Ans: C

1. A nurse is leading a word-quiz game with a group of nursing home residents because the nurse knows this activity will assist the residents in maintaining: A) Fluid intelligence B) Adaptive thinking C) Crystallized intelligence D) Psychomotor memory

Ans: C

10. A group of community health nurses is using the Stages of Change model as the foundation of a new health promotion campaign for older adults. What goal for the participants are the nurses likely to promote when working with older adults in the program? A) A recognition of the importance of screening for common health problems B) Increased participation in exercise programs and an awareness of the relationship between exercise and wellness C) The replacement of participants' unhealthy behaviors with healthy behaviors D) An awareness of the differences between life expectancy and active life expectancy

Ans: C

10. A nurse has been caring for an 83-yearold resident of a nursing home for 2 years and has developed a high level of trust with the resident. Which of the following recent changes in the resident's behavior may signal the possibility of hearing loss? A) The resident's statements occasionally suggest that he is not oriented to time. B)The resident had a recent episode where he became visibly angry at a nursing assistant. C) The resident's attention span is short and he is easily distracted. D) The resident has become increasingly disagreeable and terse in his demeanor

Ans: C

10. An older adult who appears to be between 85 and 95 has been brought to the emergency department by emergency medical services after being found wandering in the street. The older adult is filthy, confused, and exhibits numerous bruises to the face and neck as well as signs of malnutrition and dehydration. What problem should the nurses prioritize for assessment and intervention? A) Hygiene B) Malnutrition C) Dehydration D) Potential elder abuse

Ans: C

11. A nurse at the aging center organizes exercise classes including tai chi. Which of the following principles is the nurse incorporating? A) Disease prevention B) Environmental modification C) Health promotion D) Spiritual awakening

Ans: C

11. A nurse in a hospital setting assesses an older adult and is unsure if the assessment data warrant notification to the authorities for elder abuse. Which action is most appropriate for the nurse at this time? A) Determine if the person has dementia. B) Discuss findings with the family. C) Follow the hospital protocol for reporting. D) Question the visitors.

Ans: C

11. A nurse in the long-term care facility assesses an 86-year-old woman who has recently become lethargic and difficult to arouse. Her vital signs are all stable and within normal limits. Her breath sounds are diminished. Which action by the nurse should be the priority? A) Call the family and give them an update. B) Place her on high fall risk precautions. C) Send her to the emergency department. D) Tell the aides to keep an eye on her.

Ans: C

12. A nurse assesses older adults at a pulmonary clinic. Which of the following questions might best assist identify those at risk for pulmonary disorders? A) "Do any of your children smoke?" B) "In what state did you grow up?" C) "What type of job did you have?" D) "Where do you exercise?"

Ans: C

12. A wellness center nurse teaches a class of older adults about healthy habits. Which of the following interventions will make a difference in the clients' lives and as such be included by the nurse? A) Avoid alcohol consumption. B) Avoid fried foods and red meats. C) Avoid secondhand smoke. D) Avoid sunlight.

Ans: C

12. The Functional Consequences Theory approach to hearing loss identifies health promotion interventions for promoting hearing wellness. Which of the following interventions will most directly affect auditory health of the older adult? A) Avoidance of medications B) Regular colonics C) Smoking cessation D) Ear muff use in winter

Ans: C

13. A graduate nurse expresses an interest in focusing future study to healthy aging. Toward which of the following resources should the experienced nurse steer the graduate nurse? A) American Heart Association B) Healthy Brain Initiative C) Healthy People 2020 D) Springer Publishing Company

Ans: C

13. A nurse administers medications to older adults in a long-term care facility. Which of the following actions is most appropriate when the client with dementia is newly prescribed an antimuscarinic agent for urge urinary incontinence? A) Administer with a full glass of water B) Assess the client for drooling and diarrhea C) Monitor the client closely for worsening cognitive impairment D) Toilet the client before administering the medication

Ans: C

13. A nurse interviews an older adult with pulmonary disease. The client states, "I worked hard all my life in the shipyard, I provided for my family. I never smoked, why did I get this disease?" Which response by the nurse is best? A) "It is a good thing that you never smoked." B) "Pulmonary disease can happen to anyone." C) "The work in the shipyard put you at risk." D) "You feel like you are being punished..."

Ans: C

14. A nurse discusses health promotion with a group of older adult women. The nurse suggests a Pap test every 2 years. One woman states, "I haven't had a Pap since the change, why would I do that now?" Which response by the nurse is best? A) Annual checkup by your primary care provider to examine your ovaries is appropriate. B) Avoidance doesn't help with diagnosis of cancers. C) Risk of death from ovarian cancer is twice as high among women aged 65 and older. D) Women who don't have sex are at higher risk for ovarian cancer

Ans: C

15. A nurse admits an older adult from a longterm care facility into the hospital for respiratory infection. Which diagnostic testing should the nurse anticipate? A) Electrocardiogram B) Lung cancer screening C) Mantoux testing D) Pulmonary function testing

Ans: C

15. A nurse plans discharge for an older woman from China who is living with her son. Which of the following should be included in this plan to indicate that the nurse understands cultural influences? A) Include all family members in discharge planning. B) Ensure that the discharge plan does not include any technologic aspects. C) Review the discharge plans with the client and her son. D) Speak only with the client.

Ans: C

15. An older adult states, "I just feel so full so fast, I can't eat any more." Which of the following responses is most appropriate? A) "All of us feel that way after a meal." B) "Make an appointment with your health care provider." C) "Slower emptying of your stomach may be the cause." D) "This happens when you have gall stones."

Ans: C

2. A nurse presents at a conference regarding functional consequences related to urinary elimination. Which of the following statements should the nurse include? A) "Most older women will develop urinary incontinence by the age of 85." B) "Most older adults will experience hypertrophy and relaxation of muscles in the urinary tract and pelvic floor." C) "Excretion of penicillin and cimetidine are decreased in older adults." D) "Healthy older adults experience an increase in glomerular filtration rate."

Ans: C

2. An 82-year-old client is getting advice from a family member on how to drive safely. What piece of advice should the older adult follow? A) "Avoid modifying your vehicle with devices that were not supplied by the manufacturer." B) "Realize that normal, age-related changes should not affect your ability to drive safely." C) "You can consider timing your medications to avoid their interfering with safe driving." D) "You should transition from driving to using public transportation as soon as possible."

Ans: C

3. A 70-year-old tells a nurse, "I am worried that I'm losing my mind, I have difficulty remembering names as well as I used to, and I missed two health care appointments in the past month because I forgot about them." The nurse initiates a memory training program, although the nurse has been unable to identify any risk factors that might affect the older adult's cognitive abilities. Which of the following questions is the best approach to evaluating the effectiveness of the memory training program? A) "Have you seen an improvement in your memory?" B) "Are you less worried about your memory now?" C) "How have the memory training techniques helped you?" D) "Are you using the memory training techniques now?"

Ans: C

3. A nurse is teaching a group of hearingimpaired nursing home residents about hearing aids. Which of the following points should the nurse emphasize? A) It is not necessary to use the hearing aid for one-on-one conversations. B) The hearing aid should be used only in the dining room or social area. C) While inserting the hearing aid, make sure the volume is turned off. D) If whistling is heard, the volume of the hearing aid may need to be increased

Ans: C

3. An 87-year-old woman has a history of depression and hypothyroidism. She was recently diagnosed as having breast cancer. Her daughter tells her health care provider that her mother cannot participate in decision making about her care because she is too old. Which of the following statements is true about decision-making capacity? A) Determination of decision-making capacity is based on the older adult's diagnosis and chronologic age. B) The older adult has decision-making capacity if she understands most of the risks and benefits of medical treatment. C) The older adult needs to understand the issues involved in decision making and communicate about them. D) Decision-making capacity of older adults is always determined by a mental health judge.

Ans: C

4. A nurse is teaching an older adult's family about the concept of caregiver burden. Which of the following points is priority for the nurse to communicate to the family? A) "Don't feel guilty about having to hire help. Most older Americans' care is currently provided by professionals and formal services." B) "If you do eventually feel overburdened, moving your loved one to a nursing home will provide you with relief." C) "You'll find it difficult to provide for your loved one's needs if you yourself don't have a strong support system." D) "You'll actually find that for you, the benefits of providing for your loved one outweigh the negative consequences."

Ans: C

6. A nurse completes the admission assessment of an 84-year-old client to the long-term care facility. Which assessment finding would direct the nurse to document a deficit in the client's ADLs? A) The client experiences chronic pain as a result of rheumatoid arthritis. B) The client is able to ambulate with a wheeled walker for 60 ft but then requires a rest break. C) The client is able to wash self but requires assistance entering and leaving the bathtub. D) The client is unable to explain the rationale for each of the prescribed medications.

Ans: C

6. A nurse who works with older adults is teaching a colleague about the similarities and differences between gerontology and geriatrics. Which of the following questions best conveys the focus of gerontology? A) "How can we secure more funding for research and development of drugs specifically for older adults?" B) "How can we teach older adults about the relationship between their lifestyle and their health?" C) "How can we help older adults maintain wellness as they age?" D) "How can we reduce the incidence of falls among older adults who live in care facilities?"

Ans: C

6. An older adult is brought to the community clinic by an adult child with the concern of increasingly frequent lapses in memory. Which assessment question is most likely to identify potential risk factors for impaired cognitive functioning? A) "What did your mother and father die of?" B) "What line of work were you in?" C) "What medications are you currently taking?" D) "Where are you currently living?"

Ans: C

7. A nurse admits an 81-year-old man to the hospital with aspiration pneumonia. Which of the following risk factors should the nurse predict that the client has in his history? A) Cigarette smoking B) Lung cancer C) Dysphagia D) Sleep apnea

Ans: C

7. A nurse counsels an older adult regarding nutritional requirements. Which of the following teaching points is priority when discussing age-related changes in nutritional requirements? A) "If possible, try to eliminate animal fats from your diet." B) "You should try to eat less meat and proteins than you did when you were younger." C) "Overall, you don't need to take in as many calories as you used to." D) "As an older adult, you don't need to eat as many starches and complex carbohydrates."

Ans: C

7. A nurse was recently assisting an 84-year-old resident of a nursing home with the resident's biweekly bath. While the nurse was helping the resident transfer out of the bathtub, the resident grabbed on to the nurse forcefully, became rigid, and exclaimed, "Help me quick," despite the fact that the nurse was performing a safe and controlled transfer. Why might this resident have exhibited sudden anxiety during the transfer? A) The resident may be developing a cognitive deficit. B) The resident is experiencing age-related changes. C) The resident may have a fear of falling. D) The resident is ensuring safety.

Ans: C

8. A 69-year-old cigarette smoker asks the nurse questions about the potential benefits of quitting smoking, a subject avoided in past interactions. The nurse asks the client, "Would you like to quit smoking?," to which the client replies, "I will give it some serious thought." What stage of the Stages of Change model is the client demonstrating? A) Precontemplation B) Preparation C) Contemplation D) Action

Ans: C

8. A nurse manager of the long-term care facility develops plans to reduce nutritional deficits. Which of the following interventions is appropriate to include in the plan? A) Encourage residents to eat in their rooms to minimize distractions. B) Offer four to five small meals a day rather than three larger meals. C) Promote oral care for residents multiple times each day. D) Provide incentives for residents to eat all the food on their trays.

Ans: C

8. A nurse who works in a palliative setting is aware of the need to facilitate a "good death" for as many clients as possible. Which of the following interventions should be included? A) Discuss openly and explicitly the client's strengths and weaknesses. B) Ensure that a minimum of nursing interventions are performed. C) Empower the client and family to maintain as much control as possible. D) Emphasize spiritual needs rather than physical comfort and medical needs

Ans: C

8. An 81-year-old has been living for the past 2 years in a long-term care facility. However, financial pressures have required that the resident move in with the oldest child and spouse. Which of the following statements if made by the child's spouse should signal a potential risk for elder abuse? A) "I sure hope that we'll qualify for some home care because this seems pretty overwhelming." B) "This won't be easy for anyone. I think I might even end up having to juggle my work schedule." C) "He's used to being waited on here, but at our place he's going to have to fend for himself." D) "I'm probably going to even have to get some friends or neighbors to help out from time to time."

Ans: C

9. A 78-year-old client states, "I often have dry eyes, it is bothersome and irritating." What intervention should the nurse recommend? A) Daily rinses with tap water B) A medication vacation to determine if medications are the cause C) Use of over-the-counter artificial tears D) Keeping eyes closed for 3 to 5 minutes each hour

Ans: C

9. A nurse evaluates the plan of care for a client who experienced an ischemic stroke. Which of the following assessment findings should signal the nurse to the possibility that the client has developed dysphagia? A) The client complains of being excessively hungry. B) The client drinks large amounts of water with meals. C) The client pockets food in the affected cheek during meals. D) The client prefers to sit in a high Fowler's position after eating.

Ans: C

9. A nurse providing care in a long-term care setting is aware that the cognitive function of older adults does not necessarily decline on an inevitable trajectory. Which action has the greatest potential to enhance the cognitive function of residents and prevent cognitive declines? A) Encourage older adults to openly express their emotions and opinions. B) Provide residents with four or five low-fat, high-protein meals during the day. C) Encourage older adults to participate in mentally stimulating activities. D) Present older adults with numerous opportunities to make autonomous decisions.

Ans: C

9. An 81-year-old adult suffered an ischemic stroke 6 days ago. The client has failed to regain consciousness since the event. The care team has approached the client's family to obtain their views on inserting a feeding tube. Which of the following documents will allow the family to make a decision on the parent's behalf? A) A do not resuscitate (DNR) order B) A living will C) A durable power of attorney for health care D) A will

Ans: C

1. A nurse on a geriatric medical care unit consults hospice for a client. Which of the following nursing interventions should the nurse anticipate after the client begins hospice care? A) Administering chemotherapy to a client with a diagnosis of pancreatic cancer B) Assessing the deep tendon reflexes of a client with neurologic impairment C) Infusing total parenteral nutrition to a client with dysphagia D) Providing an opioid analgesic to a client with bone metastases

Ans: D

10. A series of transient ischemic attacks have caused an older adult to become dysphagic. Despite failing a swallowing assessment, the client is opposed to eating a minced and pureed diet and wishes to eat a regular diet. How should the care team respond to this request? A) Insert a feeding tube to provide nutrition while eliminating the risk of aspiration. B) Continue providing a minced and pureed diet to the client in order to ensure safety. C) Defer responsibility for feeding to the client's friends and family. D) Provide the client's requested diet after ensuring the client understands the risks.

Ans: D

11. An older adult client with urosepsis has become nonresponsive. The nurse is to identify the appropriate person to sign the consent forms for an invasive medical procedure. Which of the following actions by the nurse is appropriate? A) Find the older adult's family member to sign the consent. B) Inform the health care provider that no consent can be obtained. C) Move forward with guardianship as the client is incompetent. D) Review the chart for a health care power of attorney.

Ans: D

12. A nurse assesses an 82-year-old client who has a history of coronary artery bypass surgery and heart failure. In the interview, the family expresses concern because the client's "ability to figure out what is going on" has deteriorated. However, the client remains wise and continues to give solid life advice. Which theory explains this phenomenon? A) Crystallized intelligence declines with age. B) Cognitive skills of older adults are better than younger adults under some conditions. C) Mild cognitive impairment begins with cognitive dissidence. D) Cognitive abilities may be impaired by the client's cardiovascular disease

Ans: D

12. A nurse working for human services visits a long-term care facility. Which resident assessment finding indicates poor quality care? A) BMI of 29 B) Indentured mouth C) Serum albumin of 3.5 D) Unintentional weight loss

Ans: D

13. A nurse assesses an 85-year-old client and finds bruises on the arms and shins and a skin tear on the right hand. Which action is the priority for further nursing assessment? A) Consider the family as a reliable source of information. B) Determine if the person is depressed. C) Follow the protocol for reporting elder abuse. D) Review the client's medications and medical diagnoses

Ans: D

13. The home nurse assesses a frail older adult for fall risk using the Timed Up and Go (TUG) test. Which score places this client at high risk for falls in his home? A) 6 B) 9 C) 12 D) 15

Ans: D

14. A 90-year-old client discusses her life review with a nurse and shares information about how she has raised five children and had "ups and downs" with all of them, but overall feels satisfied with her life. Based on Cohen's empowering model, which of the following statements is the client likely to make? A) "I would sum it up this way." B) "I really would like to see the Grand Canyon." C) "I hope to learn how to Skype with my grandchildren." D) "I know I've done the best that I can do, and I expect I will continue to help my family."

Ans: D

14. A nurse discusses the future with an older adult who has had surgery for a fractured hip and is also diagnosed as having depression. The client wavers between wanting to give up and going to rehabilitation. Which of the following questions should the nurse ask to assist the client toward values clarification? A) Which pain medication works best for you? B) How can we best assist you with your activities of daily living? C) Do you prefer to bath in the morning or in the evening? D) What goals do you have for the next year?

Ans: D

14. A quality care nurse assesses the care given by a hospice. Which of the following statements by the client best reflects dignified end-of-life care? A) "I'm glad that my family is making all the decisions; it's too much for me." B) "I'm not ready to die yet; I've got a few more in me." C) "It is fine sharing a room; I like the company." D) "They listened to me and stopped the therapy."

Ans: D

14.A home care nurse evaluated the plan care for the older woman with urge incontinence. Which of the following statements by the client indicates the need for further teaching? A) "I drink enough water, but do it early in the day." B) "I make certain I don't get constipated." C) "I purchased a fancy commode for my bedroom." D) "I still have to get up two times each night to urinate."

Ans: D

15. A home care nurse assesses the home environment of an older adult client. Which of the following environmental conditions positively affects the functioning and quality of life for the client? A) The client has thick shag carpeting in the home. B) The client shares a bathroom with a teenager. C) The client's 2-year-old great grandchild plays in the living room. D) The client's home has large south-facing windows with blinds.

Ans: D

15. A home care nurse teaches a caregiver about the care of hearing aids. Which of the following statements, if made by the caregiver, indicates that further teaching is required? A) "I lay a towel over the table while working on them." B) "I turn off the aid before I change the battery." C) "I wash the earmold with warm soapy water each week." D) "I have purchased enough batteries to last a year."

Ans: D

16. An older adult develops diarrhea. Which of the following is the priority intervention for the nurse? A) Assess for pancreatitis. B) Determine the last bowel movement. C) Review meal preparation techniques with the client. D) Review the client's medication list.

Ans: D

2. A nurse discusses advance medical directives with a group of older adults at the senior citizens' center. Which of the following statements made by a member of the group indicates a need for further teaching about medical directives? A) "Advance directives address the person's right to refuse medical treatment." B) "It is helpful to see an attorney before completing a durable power of attorney." C) "Advance directives provide legal assurances that a person's preferences will be considered." D) "A durable power of attorney cannot be initiated before a person is incapacitated."

Ans: D

2. A nurse is providing an educational program about age-related macular degeneration (AMD) to a group of older adults. Which of the following statements by an older adult indicates the need for further teaching? A) "Smoking is a risk factor for AMD." B) "Macular degeneration causes a loss of central vision." C)"People with macular degeneration should have any sudden changes evaluated." D) "The dry type of macular degeneration occurs rapidly."

Ans: D

2. A nurse is teaching older adults at a senior center how to reduce the incidence of falls. Which of the following statements indicates that the nurse's teaching has been effective? A) "Benadryl is a safe medication to take for sleep." B) "It is safe to have rugs in my kitchen and bathroom." C) "It is safe to take a low dose of Ativan when I am anxious." D) "I understand that over-the-counter medications can cause falls."

Ans: D

2. A nurse teaches older adults about nutrition. Which of the following statements shows the nurse that the older adult requires further teaching? A) "Alcohol intake will interfere with absorption of B-complex vitamins and vitamin C." B) "Certain 'fluid' pills can decrease the potassium level in my blood." C) "Anticholinergic medications can cause my intestines to work slower." D) "My over-the-counter beta-carotene pill is appropriate for long-term use."

Ans: D

2. A nurse works with a program that performs interviews, blood work, and digital rectal examinations aimed at identifying older men with benign prostatic hyperplasia (BPH). The program also facilitates bathroom alterations in older adults' homes to ensure men with BPH have easy access to a toilet. Which of the following components of health promotion has yet to be implemented in the program? A) Screening B) Risk assessment C) Environmental modification D) Risk-reduction interventions

Ans: D

2. Which of the following is true about cognitive impairment and abuse of older adults? A) Older adults who live alone are always willing to acknowledge their impairments. B) Cognitively impaired older adults are usually able to meet minimum standards of care. C) When the older adult denies cognitive impairment, the risk for abuse declines. D) Older adults become more vulnerable to abuse because of cognitive impairment.

Ans: D

4. An older woman of Filipino heritage has been having rectal bleeding for several months. Her physician has told the woman and her daughters that she has advanced colon cancer. Her daughters want to obtain hospice services but the client is reluctant and does not want to discuss what she feels is "beyond her control." Which of the following concerns is this client most likely experiencing? A) Entrenched optimism in health care providers B) Individual autonomy regarding end of life C) Impaired cognition secondary to cancer D) Cultural taboo to discuss death

Ans: D

4. As part of a functional assessment, a nurse is assessing an older adult's ADLs and instrumental activities of daily living (IADLs). What piece of assessment data would most likely be considered an IADL rather than an ADL? A) The older adult is able to ambulate to and from the bathroom at home. B) The older adult can feed herself independently. C) The older adult can dress in the morning without assistance. D) The older adult is able to clean and maintain her own apartment.

Ans: D

4. Which of the following statements is true about the laws of mandatory abuse reporting? A) Government agencies, not individual nurses, are responsible for reporting abuse. B) Mandatory reporting laws require reporters to know whether abuse or neglect has occurred, rather than just suspecting it has occurred. C) The use of an abuse reporting protocol replaces individual responsibility for reporting. D) A registered nurse is mandated to report abuse or neglect if it is suspected.

Ans: D

5. A nurse administers medications to an older man. Which of the following statements if made by the client indicates understanding of the use of tamsulosin? A) "I am so happy that this medication is working to decrease my urinary incontinence." B) "I now have had much less bladder pain and cramping." C) "My blood pressure has been higher since taking this medication." D) "My urine flow starts much faster now."

Ans: D

5. A nurse teaches a health education class for older adults about constipation. Which of the following points should the nurse stress? A) Older adults who do not have a daily bowel movement should use a laxative. B)Older adults should limit their intake of highfiber foods because of a risk of lactose intolerance. C)If older adults need a medication to promote bowel regularity, a laxative or enema should be given. D)If older adults need a medication to promote bowel regularity, a bulk-forming agent is needed daily

Ans: D

5. Which of the following circumstances would be most likely to render a screening program unnecessary? A) Treatment of the disease is available at low cost. B) The disease follows a predictable course. C) The disease is more common among older adults than among younger and middle-aged adults. D) The symptoms of the disease appear at the same time that it is detectable by screening.

Ans: D

6. A 79-year-old woman is scheduled to undergo hip replacement surgery after a fracture that was caused by a fall. Which of the following age-related changes may have contributed to the woman's susceptibility to bone fracture? A) Increased protein synthesis B) Infections within the synovial capsules of the knees and ankles C) Loss of neural control of balance D) Increased bone resorption

Ans: D

6. A nurse assesses an older adult in the assisted living facility who has presbyphagia. Which of the following systems should the nurse auscultate? A) Abdomen for bruit B) Bowel sounds C) Heart tones D) Lung sounds

Ans: D

6. A nurse has noted that most of the residents who live at the long-term care facility require corrective lenses of some type. Which of the following age-related changes contributes to the loss of visual acuity? A) Decreased size and density of the lens B) Increased intraocular pressure C) Presence of floaters in the vitreous D) Decreased responsiveness of the ciliary body

Ans: D

6. A nurse who works with the older population is aware that elder abuse takes many forms. Which of the following examples most clearly constitutes elder abuse? A) A paid caregiver cleans and assists with shopping for an older adult who lives alone. B) An older adult assists with child care in exchange for room and board at her niece's house. C) A daughter manages her mother's finances after the older adult granted her power of attorney. D) A daughter changes her mother's incontinence brief only after the urine has soaked through all her clothing because she wants to save money

Ans: D

7. A 79-year-old client has been admitted to a long-term care facility because of the progression of Alzheimer disease from mild to the moderate stage. How should the nurse proceed with functional assessment? A) Document the fact that it is not possible to accurately gauge the woman's ADLs. B) Obtain assessment data from the woman's family members and friends. C) Perform assessment passively by observing and recording the woman's behavior and actions over the next several days. D) Use an assessment tool that is specifically designed for use with cognitively impaired clients.

Ans: D

7. A nurse discusses recent changes with a 74-year-old client. The client is distraught stating, "I forgot an important appointment; and I lost my wallet!" The older adult has always cherished being intelligent, alert, and informed, so even minor lapses in cognition are a source of stress. How should the nurse best interpret these recent deficits in memory? A) The older adult is likely experiencing the early stages of Alzheimer disease. B)The older adult is likely experiencing a temporary state of delirium that will self-resolve. C) The older adult may be experiencing age-related changes in personality. D) The older adult may be experiencing mild cognitive impairment.

Ans: D

8. A 68-year-old client has a long history of poor eating habits and low activity levels. The client now has a diagnosis of type 2 diabetes mellitus. Which of the following nursing interventions should be the priority? A) Adherence to diabetes screen protocols B) Education about the role that his lifestyle has played in his diagnosis C) Maintenance of function and activities of daily livings D) Self-care measures to aid in the management of his disease

Ans: D

8. A nurse working in an acute care for elders unit observes that a client on the unit frequently stumbles when ambulating with a walker. Which action by the nurse is best? A) Provide a wheelchair for the client to use for the duration of the hospital stay. B) Ask the client to remain in bed as much as possible and teach the client about falls risks. C) Place a chair in the hallway so the client can take a rest break when feeling unsteady. D) Ensure that the woman's mobility is assessed and the appropriate assistive device is provided.

Ans: D

8. In which of the following situations would a living will provide clear direction to the care and treatment of the individual involved? A) Mr. Penny, age 81, has been diagnosed with bone cancer, is experiencing severe pain, and has been presented with treatment options. B) Ms. Jelic, age 78, has been brought to the emergency department after falling on an escalator. C) Mrs. Kerr, age 77, has been admitted to hospital with an electrolyte imbalance secondary to an accidental overdose of diuretics. D) Mr. Jimenez, age 84, has suffered a severe hemorrhagic stroke and is unconscious and unlikely to survive.

Ans: D

8. The incidence of hearing loss in a longterm care facility is high, especially among white men. What strategy should care providers adopt when communicating with older adults who have hearing loss? A)Use less complex concepts when communicating with hearing-impaired older adults. B) Use a high, consistent tone and pitch when speaking to adults with hearing loss. C)Speak at a high volume directly into the less affected ear when talking to an older adult with a hearing deficit. D)Make eye contact before and during a conversation with hearing-impaired adults.

Ans: D

9. A 79-year-old experienced a severe stroke several days ago. The client's spouse has been told by the care team that he is unlikely to survive more than a few days and that aggressive treatment would likely be futile. The nurse has just entered the client's room to find the spouse softly crying at the bedside, and makes no attempt to acknowledge the nurse's presence. What is the nurse's most therapeutic response to the client's wife? A) "Do you feel like he was able to live a full life?" B) "Did you feel like you were able to discuss his treatment options thoroughly?" C) "What is it that makes you the saddest about your husband's situation?" D) "I am here; should I leave you alone for now?"

Ans: D

9. A 79-year-old man was admitted to the hospital for knee arthroplasty (replacement) due to osteoarthritis. During recovery, he developed postoperative pneumonia and became incontinent of urine while recovering from this serious infection. While being treated on the acute medicine unit, he remained in bed for several days. This client's urinary incontinence and other health challenges are most likely to result in what nursing diagnosis? A) Social isolation B) Disturbed body image C) Anxiety D) Impaired skin integrity

Ans: D

9. A gerontological nurse who works in a public health setting has limited funding for initiatives. Which of the following prevention and health promotion initiatives is most likely to result in significant benefits for the older adults who participate? A) An awareness program that promotes screening sigmoidoscopy B) Teaching older adults about falls prevention in the home C) A program of bone density screening for older adults D) An exercise program for older adults who live in the community

Ans: D

9. A palliative care team has taken over primary responsibility for the care of an older adult who has recently experienced a stroke. A visitor asks, "What is palliative care?" Which of the following is the best response by the nurse? A) "Spiritual and psychosocial care that takes place near the end of life" B) "Nursing care and medical treatment that prioritizes the wishes of patients and families" C) "The prioritization of complementary and alternative measures over biomedical interventions" D) "The provision of holistic care to patients experiencing incurable health states"

Ans: D

A nurse gives a presentation regarding eye health at a wellness clinic. Which of the following interventions should the nurse include in the teaching? A) Avoid reading under halogen lights B) Cardiovascular exercise three times a week C) Get 8 to 10 hours of sleep each night D) Wearing sunglasses with UV-absorbing lenses

Ans: D


Related study sets

Chapter 3: Inflammation, the Inflammatory Response, and Fever

View Set

Hypothesis Testing Practice Questions

View Set

Management 371 2nd Energizer (CH. 4, 5, 6.)

View Set

TC 472. MISCELLANEOUS PROVISIONS

View Set

Genetics Chapter 19: Gene Mutation, DNA Repair, and Recombination

View Set

Business to the Rescue Unit Review

View Set